PT18.S2.Q6 - A study was designed to establish which effect...

Ashley2018-1Ashley2018-1 Live Member
edited May 2021 in Logical Reasoning 2249 karma

Aren't there two main ways to weaken an argument? Either by going for the premises (contradicting them) or showing why the conclusion doesn't necessarily follow from them? I thought C did the first, but now I am having doubts. The stimulus concludes that oil rigs have no adverse (or harmful) effect on wildlife because when the areas near oil rigs and control sites several miles away were compared, no significant differences were found. When C mentions there was "contamination from sewage and industrial effluent," could I assume those pollutants came from the oil rigs or not? And does this choice contradict the premise that there were differences between the sites near the oil rigs and control?

Admin Note: https://7sage.com/lsat_explanations/lsat-18-section-2-question-06/

Comments

  • cwrussell42cwrussell42 Member
    17 karma

    C would weaken the researchers' conclusion if it were true, but it is contradicted by the stimulus. We know that the researchers found no differences between sea-bottom near rigs and sea-bottom several miles from any rig, so how could the rigs be reducing species diversity or animal life density and still be identical to the sites far away from any rig?

  • canihazJDcanihazJD Alum Member Sage
    8313 karma

    @"Na-leh2021" said:
    Either by going for the premises (contradicting them)...

    We don't attack premises. They are accepted as true. Our workspace is the premise to conclusion relationship.

  • Cynthia-2Cynthia-2 Member
    edited May 2021 498 karma

    I have never gotten a weakening question wrong and the reason for this is that if you attack the premise, you will always get the wrong answer. The trick is to attack the premise - conclusion relationship. What I do is re-word the conclusion, in this case, it says "that oil rigs have no adverse effect", say that it DOES have an effect and choose the answer that states that. In this case B states that NO , it does have an effect, the oil drifted to another place. If you always look the opposite of what is stated, youll get the right answer each time. GL

Sign In or Register to comment.